In a study of 24 criminals convicted of antitrust offenses, the average age was 60 years, with a standard deviation of 7.4 years. Construct a 95% confidence interval on the true mean age. (Give your answers correct to one decimal place.)___ to____ years

Answers

Answer 1

Answer: 56.9 years to 63.1 years.

Step-by-step explanation:

Confidence interval for population mean (when population standard deviation is unknown):

[tex]\overline{x}\pm t_{\alpha/2}{\dfrac{s}{\sqrt{n}}}[/tex]

, where [tex]\overline{x}[/tex]= sample mean, n= sample size, s= sample standard deviation, [tex]t_{\alpha/2}[/tex]= Two tailed t-value for [tex]\alpha[/tex].

Given: n= 24

degree of freedom = n- 1= 23

[tex]\overline{x}[/tex]= 60 years

s= 7.4 years

[tex]\alpha=0.05[/tex]

Two tailed t-critical value for significance level of [tex]\alpha=0.05[/tex] and degree of freedom 23:

[tex]t_{\alpha/2}=2.0687[/tex]

A 95% confidence interval on the true mean age:

[tex]60\pm (2.0686){\dfrac{7.4}{\sqrt{24}}}\\\\\approx60\pm3.1\\\\=(60-3.1,\ 60+3.1)\\\\=(56.9,\ 63.1)[/tex]

Hence, a 95% confidence interval on the true mean age. : 56.9 years to 63.1 years.


Related Questions

What is the length of AD
A.17
B.15
C.7
D.1

Answers

Answer:

15 units

Step-by-step explanation:

Point D is at 8

Point A is at -7

D - A

8 - -7

8+7

15 units

Answer:

the answer is 15

you can just count the number of steps going forward from -7 to 8.

A man starts repaying a loans with first insfallameny of rs.10 .If he increases the instalment by Rs 5 everything months, what amount will be paid by him in the 30the instalment.​

Answers

Answer:

30×5=150

so 150+10=160

thus his payment in the 30th installment is

rs.160

(2+1/2) (2^2-1+1/4) find the expression in the form of cubes and differences of two terms.​

Answers

Answer:

Consider the following identity:

a³ - b³ = (a + b)(a² - ab + b²)

Let a = 2, b = 1/2

(2 + 1/2)(2² - 2*1/2 + 1/2²) = 2³ - (1/2)³ =8 - 1/8

Use the algebraic identity given below

[tex]\boxed{\sf a^3-b^3=(a+b)(a^2-ab+b^2)}[/tex]

[tex]\\ \sf\longmapsto (2+\dfrac{1}{2})(2^2-1+\dfrac{1}{4})[/tex]

[tex]\\ \sf\longmapsto (2+\dfrac{1}{2})(2^2-2\times \dfrac{1}{2}+\dfrac{1}{2}^2)[/tex]

Here a =2 and b=1/2

[tex]\\ \sf\longmapsto 2^3-\dfrac{1}{2}^3[/tex]

[tex]\\ \sf\longmapsto 8-\dfrac{1}{8}[/tex]

Instructions: Determine if the two triangles in
the image are congruent. If they are, state how
you know by identifying the postulate.
th

Answers

The 2 triangles are congruent

Given the formula a=b+c, rewrite the formula to solve for c

Answers

9514 1404 393

Answer:

  c = a - b

Step-by-step explanation:

Subtract b from both sides of the equation.

  a = b + c

  a - b = b + c - b . . . . . show the subtraction

  a - b = c . . . . . . . . . . simplify

  c = a - b . . . . . . write with c on the left

Which expression simplifies to 7W+5?
. – 2w + 3 + 5W – 2
C. -3w + 5(2W + 1)
Cual es la respuesta

Answers

Answer:

[tex]\large \boxed{\mathrm{-3w + 5(2w + 1)}}[/tex]

Step-by-step explanation:

-2w + 3 + 5w - 2

Combine like terms.

3w + 1

-3w + 5(2w + 1)

Expand brackets.

-3w + 10w + 5

Combine like terms.

7w + 5

Answer:

The answer is C.

-3w +5(2w +1)

Step-by-step explanation:

Y= 3x-1
2x+6=y substitution method

Answers

Answer: (7, 20)

Concept:

There are three general ways to solve systems of equations:

EliminationSubstitutionGraphing

Since the question has specific requirements, we are going to use substitution to solve the equations.

Solve:

Given equations

y = 3x - 1

2x + 6 = y

Substitute the y value since both equations has isolated [y]

2x + 6 = 3x - 1

Add 1 on both sides

2x + 6 + 1 = 3x - 1 + 1

2x + 7 = 3x

Subtract 2x on both sides

2x + 7 - 2x = 3x - 2x

[tex]\boxed{x=7}[/tex]

Find the value of y

y = 3x - 1

y = 3(7) - 1

y = 21 - 1

[tex]\boxed{y=20}[/tex]

Hope this helps!! :)

Please let me know if you have any questions

find the greatest common factor of 108d^2 and 216d

Answers

Answer:

Below

Step-by-step explanation:

If d is a positive number then the greatest common factor is 108d.

To get it isolate d and d^2 from the numbers.

108 divides 216. (216 = 2×108)

Then the greatest common factor of 216 and 108 is 108.

For d^2 and d we will follow the same strategy

d divides d^2 (d^2 = d*d)

Then the greatest common factor of them is d.

So the greatest common factor will be 108d if and only if d is positive. If not then 108 is the answer

Answer:

[tex]\boxed{108d}[/tex]

Step-by-step explanation:

Part 1: Find GCF of variables

The equation gives d ² and d as variables. The GCF rules for variables are:

The variables must have the same base.If one variable is raised to a power and the other is not, the GCF is the variable that does not have a power.If one variable is raised to a power and the other is raised to a power of lesser value, the GCF is the variable with the lesser value power.

The GCF for the variables is d.

Part 2: Find GCF of bases (Method #1)

The equation gives 108 and 216 as coefficients. To check for a GCF, use prime factorization trees to find common factors in between the values.

Key: If a number is in bold, it is marked this way because it cannot be divided further AND is a prime number!

Prime Factorization of 108

108 ⇒ 54 & 2

54 ⇒ 27 & 2

27 ⇒ 9 & 3

9 ⇒ 3 & 3

Therefore, the prime factorization of 108 is 2 * 2 * 3 * 3 * 3, or simplified as 2² * 3³.

Prime Factorization of 216

216 ⇒ 108 & 2

108 ⇒ 54 & 2

54 ⇒ 27 & 2

27 ⇒ 9 & 3

9 ⇒ 3 & 3

Therefore, the prime factorization of 216 is 2 * 2 * 2 * 3 * 3 * 3, or simplified as 2³ * 3³.

After completing the prime factorization trees, check for the common factors in between the two values.

The prime factorization of 216 is 2³ * 3³ and the prime factorization of 108 is 2² * 3³.  Follow the same rules for GCFs of variables listed above and declare that the common factor is the factor of 108.

Therefore, the greatest common factor (combining both the coefficient and the variable) is [tex]\boxed{108d}[/tex].

Part 3: Find GCF of bases (Method #2)

This is the quicker method of the two. Simply divide the two coefficients and see if the result is 2. If so, the lesser number is immediately the coefficient.

[tex]\frac{216}{108}=2[/tex]

Therefore, the coefficient of the GCF will be 108.

Then, follow the process described for variables to determine that the GCF of the variables is d.

Therefore, the GCF is [tex]\boxed{108d}[/tex].

The equation of a circle centered at the origin with a radius of unit length is x2 + y2 = 1. This equation changes if the center of the circle is not located at the origin or the radius is not of unit length.

Answers

Answer:

The equation for a unit radius circle, centered at the origin is:

x^2 + y^2 = 1

Now, if we want to move it horizontally, you can recall to the horizontal translations:

f(x) -----> f(x - a)

Moves the graph to the right by "a" units.

A vertical translation is similar.

Then, if we want a circle centered in the point (a, b) we have:

(x - a)^2  + (y - b)^2 = 1.

Now, if you want to change the radius, we can actually write the unit circle as:

x^2 + y^2 = 1^2

Where if we set x = 0, 1 = y, this is our radius

So if we have:

x^2 + y^2 = R^2

And we set the value of x = 0, then R = y.

So our radius is R.

Then:

"A circle of radius R, centered in the point (a, b) is written as:

(x - a)^2 + (y - b)^2 = R^2

Which decimal number is less than 1.56?

Answers

The decimal number is 0.9

work out angle pqr please if you know it i willl give brainliest to fastest person

Answers

Answer:

∠PQR =  67.36° (Approx)

Step-by-step explanation:

Given:

PRQ is a right triangle

∠R = 90°

PQ = 13 cm

QR = 5 cm

PR = 12 cm

Find:

∠PQR = ?

Computation:

Using trigonometry application.

SinФ = 12 / 13

SinФ = 0.92

Sin 67.36° = 0.92

So,

∠PQR =  67.36° (Approx)

La'Vonn rolled a die 100 times. His results are below. What is the relative frequency for La'Vonn rolling a 3?

Answers

What type of die 6 side or 12 side

Answer:

.15

Step-by-step explanation:

Need Help
Please Show Work​

Answers

Answer:

18 - 8 * n = -6 * n

The number is 9

Step-by-step explanation:

Let n equal the number

Look for key words such as is which means equals

minus is subtract

18 - 8 * n = -6 * n

18 -8n = -6n

Add 8n to each side

18-8n +8n = -6n+8n

18 =2n

Divide each side by 2

18/2 = 2n/2

9 =n

The number is 9

━━━━━━━☆☆━━━━━━━

▹ Answer

n = 9

▹ Step-by-Step Explanation

18 - 8 * n = -6 * n

Simple numerical terms are written last:

-8n + 18 = -6n

Group all variable terms on one side and all constant terms on the other side:

(-8n + 18) + 8n = -6n + 8n

n = 9

Hope this helps!

CloutAnswers ❁

━━━━━━━☆☆━━━━━━━

Put the following numbers in order from least to greatest: π/2,-4,0.09,17,√3,-1/7,√225

Answers

Answer:

-4, -1/7,0.09,π/2,√3 ,√225,17

Step-by-step explanation:

π/2, is approx 1.5

-4,

0.09,

17,

√3 is approx 1.7

,-1/7, is approx -.143

√225 = 15

From most negative to greatest

-4, -1/7,0.09,π/2,√3 ,√225,17

Answer:

[tex]-4, -1/7, 0.09, \pi/2, \sqrt3, \sqrt{225}, 17[/tex]

Step-by-step explanation:

So we have the numbers:

[tex]\pi/2, -4, 0.09, 17, \sqrt3, -1/7, \sqrt{225}[/tex]

(And without using a calculator) approximate each of the values.

π is around 3.14, so π/2 is around 1.57.

17 squared is 289, so 1.7 squared is 2.89. Thus, the square root of 13 is somewhere between 1.7 and 1.8.

-1/7 can be divided to be about -0.1429...

And the square root of 225 is 15.

Now, use the approximations to place the numbers:

[tex]\pi/2\approx1.57; -4; 0.09;17;\sqrt3 \approx1.7; -1/7\approx-0.14;\sqrt{225}=15[/tex]

The smallest is -4.

Next is -1/7 or about -0.14

Followed by the first positive, 0.09.

And then with π/2 or 1.57

And then a bit bigger with the square root of 3 or 1.7.

And then with the square root of 225 or 15.

And finally the largest number 17.

Thus, the correct order is:

[tex]-4, -1/7, 0.09, \pi/2, \sqrt3, \sqrt{225}, 17[/tex]

What is the value of this expression when x = -6 and y = — 1/2? 4(x^2+3) -2y A. -131 B. -35 C. 57 1/2 D. 157

Answers

Answer:

D

Step-by-step explanation:

[tex]4(x^2+3)-2y\\\\=4((-6)^2+3)-2(\frac{-1}{2} )\\\\=4(36+3)+1\\\\=4(39)+1\\\\=156+1\\\\=157[/tex]

The value of the expression 4(x² + 3) - 2y is 157, when x = -6 and y = -1/2.

What is an algebraic expression?

An algebraic expression is consists of variables, numbers with various mathematical operations,

The given expression is,

4(x² + 3) - 2y

Substitute x = -6 and y = -1/2 to find the value of expression,

= 4 ((-6)² + 3) - 2(-1/2)

= 4 (36 + 3) + 1

= 4 x 39 + 1

= 156 + 1

= 157

The required value of the expression is 157.

To know more about Algebraic expression on:

https://brainly.com/question/19245500

#SPJ2

Integers that are not whole numbers​

Answers

Answer:

a negative integer

Step-by-step explanation:

Find the Correlation of the following two variables X: 2, 3, 5, 6 Y: 1, 2, 4, 5

Answers

Answer:

The correlation of X and Y is 1.006

Step-by-step explanation:

Given

X: 2, 3, 5, 6

Y: 1, 2, 4, 5

n = 4

Required

Determine the correlation of x and y

Start by calculating the mean of x and y

For x

[tex]M_x = \frac{\sum x}{n}[/tex]

[tex]M_x = \frac{2 + 3+5+6}{4}[/tex]

[tex]M_x = \frac{16}{4}[/tex]

[tex]M_x = 4[/tex]

For y

[tex]M_y = \frac{\sum y}{n}[/tex]

[tex]M_y = \frac{1+2+4+5}{4}[/tex]

[tex]M_y = \frac{12}{4}[/tex]

[tex]M_y = 3[/tex]

Next, we determine the standard deviation of both

[tex]S = \sqrt{\frac{\sum (x - Mean)^2}{n - 1}}[/tex]

For x

[tex]S_x = \sqrt{\frac{\sum (x_i - Mx)^2}{n -1}}[/tex]

[tex]S_x = \sqrt{\frac{(2-4)^2 + (3-4)^2 + (5-4)^2 + (6-4)^2}{4 - 1}}[/tex]

[tex]S_x = \sqrt{\frac{-2^2 + (-1^2) + 1^2 + 2^2}{3}}[/tex]

[tex]S_x = \sqrt{\frac{4 + 1 + 1 + 4}{3}}[/tex]

[tex]S_x = \sqrt{\frac{10}{3}}[/tex]

[tex]S_x = \sqrt{3.33}[/tex]

[tex]S_x = 1.82[/tex]

For y

[tex]S_y = \sqrt{\frac{\sum (y_i - My)^2}{n - 1}}[/tex]

[tex]S_y = \sqrt{\frac{(1-3)^2 + (2-3)^2 + (4-3)^2 + (5-3)^2}{4 - 1}}[/tex]

[tex]S_y = \sqrt{\frac{-2^2 + (-1^2) + 1^2 + 2^2}{3}}[/tex]

[tex]S_y = \sqrt{\frac{4 + 1 + 1 + 4}{3}}[/tex]

[tex]S_y = \sqrt{\frac{10}{3}}[/tex]

[tex]S_y = \sqrt{3.33}[/tex]

[tex]S_y = 1.82[/tex]

Find the N pairs as [tex](x-M_x)*(y-M_y)[/tex]

[tex](2 - 4)(1 - 3) = (-2)(-2) = 4[/tex]

[tex](3 - 4)(2 - 3) = (-1)(-1) = 1[/tex]

[tex](5 - 4)(4 - 3) = (1)(1) = 1[/tex]

[tex](6-4)(5-3) = (2)(2) = 4[/tex]

Add up these results;

[tex]N = 4 + 1 + 1 + 4[/tex]

[tex]N = 10[/tex]

Next; Evaluate the following

[tex]\frac{N}{S_x * S_y} * \frac{1}{n-1}[/tex]

[tex]\frac{10}{1.82* 1.82} * \frac{1}{4-1}[/tex]

[tex]\frac{10}{3.3124} * \frac{1}{3}[/tex]

[tex]\frac{10}{9.9372}[/tex]

[tex]1.006[/tex]

Hence, The correlation of X and Y is 1.006

In the nation of Gondor, the EPA requires that half the new cars sold will meet a certain particulate emission standard a year later. A sample of 64 one-year-old cars revealed that only 24 met the particulate emission standard. The test statistic to see whether the proportion is below the requirement is

Answers

Complete Question

In the nation of Gondor, the EPA requires that half the new cars sold will meet a certain particulate emission standard a year later. A sample of 64 one-year-old cars revealed that only 24 met the particulate emission standard. The test statistic to see whether the proportion is below the requirement is:

A -1.645

B -2.066

C -2.000

D-1.960

Answer:

The  correct option is C

Step-by-step explanation:

From the question we are told that

   The  population mean is  [tex]p = 0.50[/tex]

    The sample size is  [tex]n = 64[/tex]

     The  number that met the standard is  [tex]k = 24[/tex]

       

Generally the sample proportion is mathematically evaluated   as

             [tex]\r p = \frac{24}{64}[/tex]

             [tex]\r p =0.375[/tex]

Generally the standard error is mathematically evaluated as  

       [tex]SE = \sqrt{ \frac{p(1- p )}{n} }[/tex]

=>    [tex]SE = \sqrt{ \frac{0.5 (1- 0.5 )}{64} }[/tex]

=>   [tex]SE = 0.06525[/tex]

The  test statistics is evaluated as

        [tex]t = \frac{ \r p - p }{SE}[/tex]

        [tex]t = \frac{ 0.375 - 0.5 }{0.0625}[/tex]

        [tex]t = -2[/tex]

Find the equation of a parabola that has a vertex (3,5) and passes through the point (1,13).
Oy= -27 - 3)' +5
Oy=2(x + 3) - 5
Oy=2(0 - 3)' + 5
Oy= -3(2 – 3) + 5
PLEASE HELP ME!!

Answers

Answer:

y = 2(x - 3)² + 5

Step-by-step explanation:

The equation of a parabola in vertex form is

y = a(x - h)² + k

where (h, k) are the coordinates of the vertex and a is a multiplier

Here (h, k) = (3, 5), thus

y = a(x - 3)² + 5

To find a substitute (1, 13) into the equation

13 = a(1 - 3)² + 5 ( subtract 5 from both sides )

8 = 4a ( divide both sides by 4 )

a = 2, then

y = 2(x - 3)² + 5 ← equation of parabola in vertex form

If f(4x-15)=8x-27,find f(x)?

Answers

Answer:

If we put x=17/4

f(4×17/4-15)=8×17/4-27

f(2x=34-27

f(x)=7.

Hope i helped you.

Every high school senior takes the SAT at a school in St. Louis. The high school guidance director at this school collects data on each graduating senior’s GPA and their corresponding SAT test score. The guidance director is conducting a _________ in this experimental design.

A. sample survey
B. census
C. sample poll
D. random sample

Answers

Voće Eu tudo bem yea Iliana buns iOS build. Lbvac la vaca leche 457 sample poll

The guidance director is conducting a sample poll in this experimental design.

What is sample?

Sample is a part of population. It does not comprises whole population. It is representatitive of whole population.

How to fill blank?

We are required to fill the blank with appropriate term among the options.

The correct option is sample poll because the guidance director collects data in his school only.

Census collects the whole population of the country.

Sample poll means collecting data from small population.

Random sample means collecting data from a part of popultion without identifying any variable.

Hence we found that he was doing sample poll.

Learn more about sample at https://brainly.com/question/24466382

#SPJ2

A normal population has a mean of 65 and a standard deviation of 13. You select a random sample of 25. Compute the probability that the sample mean is: (Round your z values to 2 decimal places and final answers to 4 decimal places): Greater than 69.

Answers

Answer:

0.0618

Step-by-step explanation:

z = (x - μ)/σ, where

x is the raw score = 69

μ is the sample mean = population mean = 65

σ is the sample standard deviation

This is calculated as:

= Population standard deviation/√n

Where n = number of samples = 25

σ = 13/√25

σ = 13/5 = 2.6

Sample standard deviation = 2.6

z = (69 - 65) / 2.6

z = 4/2.6

z = 1.53846

Approximately to 2 decimal places = 1.54

Using the z score table to determine the probability,

P(x = 69) = P(z = 1.54)

= 0.93822.

The probability that the sample mean is greater than 69 is

P(x>Z) = 1 - 0.93822

P(x>Z) = 0.06178

Approximately to 4 decimal places = 0.0618

Evaluate S_5 for 600 + 300 + 150 + … and select the correct answer below. A. 1,162.5 B. 581.25 C. 37.5 D. 18,600

Answers

Answer:

  A.  1,162.5

Step-by-step explanation:

Write the next two terms and add them up:

  S5 = 600 +300 +150 +75 +37.5 = 1162.5 . . . . matches choice A

Answer: Choice A 1,162.5

================================================

Explanation:

{600, 300, 150, ...} is a geometric sequence starting at a = 600 and has common ratio r = 1/2 = 0.5, this means we cut each term in half to get the next term. We could do this to generate five terms and then add them up. Or we could use the formula below with n = 5

Sn = a*(1-r^n)/(1-r)

S5 = 600*(1-0.5^5)/(1-0.5)

S5 = 1,162.5

-----------

Check:

first five terms = {600, 300, 150, 75, 37.5}

S5 = sum of the first five terms

S5 = 600+300+150+75+37.5

S5 = 1,162.5

Because n = 5 is relatively small, we can quickly confirm the answer. With larger values of n, a spreadsheet is the better option.

n a survey of a group of​ men, the heights in the​ 20-29 age group were normally​ distributed, with a mean of inches and a standard deviation of inches. A study participant is randomly selected. Complete parts​ (a) through​ (d) below. ​(a) Find the probability that a study participant has a height that is less than inches. The probability that the study participant selected at random is less than inches tall is nothing. ​(Round to four decimal places as​ needed.) ​(b) Find the probability that a study participant has a height that is between and inches. The probability that the study participant selected at random is between and inches tall is nothing. ​(Round to four decimal places as​ needed.) ​(c) Find the probability that a study participant has a height that is more than inches. The probability that the study participant selected at random is more than inches tall is nothing. ​(Round to four decimal places as​ needed.) ​(d) Identify any unusual events. Explain your reasoning. Choose the correct answer below.

Answers

Answer:

(a) The probability that a study participant has a height that is less than 67 inches is 0.4013.

(b) The probability that a study participant has a height that is between 67 and 71 inches is 0.5586.

(c) The probability that a study participant has a height that is more than 71 inches is 0.0401.

(d) The event in part (c) is an unusual event.

Step-by-step explanation:

The complete question is: In a survey of a group of​ men, the heights in the​ 20-29 age group were normally​ distributed, with a mean of 67.5 inches and a standard deviation of 2.0 inches. A study participant is randomly selected. Complete parts​ (a) through​ (d) below. ​(a) Find the probability that a study participant has a height that is less than 67 inches. The probability that the study participant selected at random is less than inches tall is nothing. ​(Round to four decimal places as​ needed.) ​(b) Find the probability that a study participant has a height that is between 67 and 71 inches. The probability that the study participant selected at random is between and inches tall is nothing. ​(Round to four decimal places as​ needed.) ​(c) Find the probability that a study participant has a height that is more than 71 inches. The probability that the study participant selected at random is more than inches tall is nothing. ​(Round to four decimal places as​ needed.) ​(d) Identify any unusual events. Explain your reasoning. Choose the correct answer below.

We are given that the heights in the​ 20-29 age group were normally​ distributed, with a mean of 67.5 inches and a standard deviation of 2.0 inches.

Let X = the heights of men in the​ 20-29 age group

The z-score probability distribution for the normal distribution is given by;

                          Z  =  [tex]\frac{X-\mu}{\sigma}[/tex]  ~ N(0,1)

where, [tex]\mu[/tex] = population mean height = 67.5 inches

            [tex]\sigma[/tex] = standard deviation = 2 inches

So, X ~ Normal([tex]\mu=67.5, \sigma^{2}=2^{2}[/tex])

(a) The probability that a study participant has a height that is less than 67 inches is given by = P(X < 67 inches)

 

      P(X < 67 inches) = P( [tex]\frac{X-\mu}{\sigma}[/tex] < [tex]\frac{67-67.5}{2}[/tex] ) = P(Z < -0.25) = 1 - P(Z [tex]\leq[/tex] 0.25)

                                                                 = 1 - 0.5987 = 0.4013

The above probability is calculated by looking at the value of x = 0.25 in the z table which has an area of 0.5987.

(b) The probability that a study participant has a height that is between 67 and 71 inches is given by = P(67 inches < X < 71 inches)

    P(67 inches < X < 71 inches) = P(X < 71 inches) - P(X [tex]\leq[/tex] 67 inches)

    P(X < 71 inches) = P( [tex]\frac{X-\mu}{\sigma}[/tex] < [tex]\frac{71-67.5}{2}[/tex] ) = P(Z < 1.75) = 0.9599

    P(X [tex]\leq[/tex] 67 inches) = P( [tex]\frac{X-\mu}{\sigma}[/tex] [tex]\leq[/tex] [tex]\frac{67-67.5}{2}[/tex] ) = P(Z [tex]\leq[/tex] -0.25) = 1 - P(Z < 0.25)

                                                                = 1 - 0.5987 = 0.4013

The above probability is calculated by looking at the value of x = 1.75 and x = 0.25 in the z table which has an area of 0.9599 and 0.5987 respectively.

Therefore, P(67 inches < X < 71 inches) = 0.9599 - 0.4013 = 0.5586.

(c) The probability that a study participant has a height that is more than 71 inches is given by = P(X > 71 inches)

 

      P(X > 71 inches) = P( [tex]\frac{X-\mu}{\sigma}[/tex] > [tex]\frac{71-67.5}{2}[/tex] ) = P(Z > 1.75) = 1 - P(Z [tex]\leq[/tex] 1.75)

                                                                 = 1 - 0.9599 = 0.0401

The above probability is calculated by looking at the value of x = 1.75 in the z table which has an area of 0.9599.

(d) The event in part (c) is an unusual event because the probability that a study participant has a height that is more than 71 inches is less than 0.05.

Solve the following system of equations for x: y = 4 -x^2
X - y =
2

Answers

Answer:

B. x=2, x=-3

Step-by-step explanation:

Hello,

We are given this system of equations:

y=4-x²

x-y=2

And we want to solve it for x.

There are a couple ways to solve systems of equations, but let's use substitution in this problem, which we will set one variable equal to an expression containing the other variable, use that expression to solve for the variable the expression contains in the other equation, then use the value of the solved variable to find the value of the other variable.

But in this case, we just need to find x, which is just one of the variables, so there's no need to find the values of y.

In x-y=2, subtract x from both sides.

-y=-x+2

Multiply both sides by -1.

y=x-2

Substitute y as x-2 in y=4-x².

x-2=4-x²

Add x² to both sides.

x²+x-2=4

Subtract 4 from both sides.

x²+x-6=0

Now factor using FOIL.

1 (coefficient in front of x) is the sum of the numbers used in the binomials, while -6 is their product.

Two numbers that add up to 1 but multiply to get -6 are -2 and 3.

So now factor:

(x-2)(x+3)=0

Split and solve:

x-2=0

x=2

x+3=0

x=-3

The solutions for x are x=2 and x=-3, which means the answer is B.

Hope this helps!

(Algebra) PLZ HELP ASAP!

Answers

Answer: Its everthing except irrational

Step-by-step explanation:

The height of a triangle is 5 yards greater than the base. The area of the triangle is 273 square yards. Find the length of the base and the height of the triangle.

Answers

Answer:

Base = 21 while Height = 16

Charlie's flower bed has a length of 4 feet and a width of four sixths feet. Which of the following is true
1 The area of the flower bed is equal to 6 square feet.
2The area of the flower bed is larger to 6 square feet.
3 The area of the flower bed is equal to 4 square feet
4 The area of the flower bed is smaller than 4 square feet.

Answers

Option 4 is correct

Answer:

Option 4) The area of the flower bed is smaller than 4 square feet.

Step-by-step explanation:

Let’s solve for the area of the flower bed.

Consider that the flower bed is a rectangle.

The area of a recrangle is given by the formula:

A = length x width

The area of the flower bed is:

4 ft x 4/6 ft = 2 2/3 ft^2

2 2/3 ft ^2 < 4 ft^2

Therefore option 4 is the correct answer.

find the length of the arc. round your answer to nearest tenth

Answers

41.9 mi

Step-by-step explanation:

First, we convert the angle from degree measure to radian measure:

[tex]\theta = 240°×\left(\dfrac{\pi}{180°}\right)= \dfrac{4\pi}{3}\:\text{rad}[/tex]

Using the definition of an arc length [tex]s[/tex]

[tex]s = r\theta[/tex]

[tex]\:\:\:\:=(10\:\text{mi})\left(\dfrac{4\pi}{3}\:\text{rad}\right)[/tex]

[tex]\:\:\:\:= 41.9\:\text{mi}[/tex]

look at the image below

Answers

that's what I got, you can confirm if it's correct or not... I'm not sure

have a nice dayʘ‿ʘ

Other Questions
can somone please highlight that makeup the appositive phrase The alien spaceship, a flying silver disk, hovered over Dory's school. To measure Monarch butterfly migration,scientist tag and release 100 butterflies inKansas and Missouri before traveling toMexico. While in Mexico, the samescientist captured another 100 butterfliesof which 15 are tagged. Based on thisinformation, how many butterflies wouldyou predict start in Kansas and Missouriand migrate to Mexico? What characteristic of life can you live without Standard rate per direct labor-hour $ 2 Standard direct labor-hours for each unit produced 3 Units manufactured 1,000 Actual direct labor-hours worked during the month 3,300 Total actual variable manufacturing overhead $ 6,600 Knowledge Check 01 Assume that direct labor-hours is used as the overhead allocation base. What is the variable overhead efficiency variance Jeremy reminded the board members that it is important to understand the inflation rate of a country before entering into business. Jeremy is concerned with the ________ risks of doing business. Which tools would I use to heat 400 mL of water In a group of 25 people, only three languages are spoken English, Spanish and German. If there is at least one person who speaks all the three languages, how many people can interact with each other in English and German? 4 people speak two languages but do not speak Spanish One fifth of the group speaks more than one language. a. Name a chemical or product that was once considered safe but is now consideredharmful. (1 point)- solve the equation(2x-5x)(4y-6x) An archer shoots an arrow in the air, horizontally. However, after moving somedistance, the arrow falls to the ground. Name and define the initial force that setsthe arrow in motion. Explain why the arrow ultimately falls down. Graph the parabola.y=x^2-4x+2 Luis, Diego, and Cecil are going fishing.Luis brings 4 cans of worms. Diego brings3 cans of worms plus 2 extra worms. Cecilbrings 2 cans of worms. If they have atotal of 65 worms and each can containsthe same number of worms, howmany worms are in each can? 5. Which period or event was led by Bartolomeu Dias, Christopher Columbus, Vasco daGama, and Ferdinand Magellan, among others?A.CrusadesC.RenaissanceB.ReformationD.Exploration Find the area of a triangle whose two sides are 12 inches and 14 inches long, and has a perimeter of 34 inches. A. 23.24 in B. 47.91 in C. 24.74 in D. 79.84 in calculate horizontal distance travelled by a ball travelling with a speed of 20root2 mper sec without hitting ceiling of height 20 m per sec Integrate[Exp[Power[sinx,2]]sin2x,x] Triangles ABC and DEF are similar. Find the missing angles. Identify the conjugate pairs in the following acid-base reactionHCO(aq) + CHN(aq) HNO-(aq) + HCHN+(aq) 6(2+5)-2^2x2(9/3)+10/5 Andrew rents rooms in his hotel for an average of $97 per night. The variable cost per rented room is $36. His fixed costs are $52,000 per month. How many rooms does he have to rent per month in order to break even?